Unique Inverse of a Modulo m When a and m are Relatively Prime

  • Thread starter Thread starter brad sue
  • Start date Start date
  • Tags Tags
    Inverse Prime
Click For Summary
If a and m are relatively prime positive integers, the inverse of a modulo m is unique. The discussion revolves around proving that if there are two solutions, b and c, to the congruence ax ≡ 1 (mod m), then b must equal c modulo m. The key step involves recognizing that from a*b ≡ a*c (mod m), one cannot simply cancel a, but instead must use the property that gcd(a, m) = 1. This leads to the conclusion that m divides a*(b-c), implying that b ≡ c (mod m). The problem is thus resolved by applying the theorem discussed in class.
brad sue
Messages
270
Reaction score
0
Hi,
I need help with this problem.

Show that if a and m are relatively prime positive integers, then the inverse of a modulo m is unique modulo m.
[hint: assume that there are 2 solutions b and c of the congruence ax==1(mod m). No need to prove that b==c (mod m) ]

I have just started:

a*b==1(mod m) and c*a==1(mod m)-->a*b==c*a(mod m)
-->b==c (mod m)
then ??
Can I have some help please?
B
 
Physics news on Phys.org
b=c (mod m) is what you're trying to prove. The problem is deriving this from a*b=a*c (mod m). You can't just cancel a like in ordinary multiplication (for example, 3*1=3*4 (mod 9), but 1\neq4(mod 9)). a*b=a*c (mod m) is equivalent to saying that m divides a*b-a*c=a*(b-c). What can you conclude from this given that a and m are relatively prime?
 
StatusX said:
b=c (mod m) is what you're trying to prove. The problem is deriving this from a*b=a*c (mod m). You can't just cancel a like in ordinary multiplication (for example, 3*1=3*4 (mod 9), but 1\neq4(mod 9)). a*b=a*c (mod m) is equivalent to saying that m divides a*b-a*c=a*(b-c). What can you conclude from this given that a and m are relatively prime?

we can say that gcd(a,m)=1 and a*b=a*c (mod m)
--> b=c(mod m) (according a theorem we saw in class)..
Am I right?
B
 
Then you're done. Given that theorem, it's a pretty trivial problem.
 
Question: A clock's minute hand has length 4 and its hour hand has length 3. What is the distance between the tips at the moment when it is increasing most rapidly?(Putnam Exam Question) Answer: Making assumption that both the hands moves at constant angular velocities, the answer is ## \sqrt{7} .## But don't you think this assumption is somewhat doubtful and wrong?

Similar threads

  • · Replies 2 ·
Replies
2
Views
1K
Replies
4
Views
1K
  • · Replies 16 ·
Replies
16
Views
3K
  • · Replies 9 ·
Replies
9
Views
3K
  • · Replies 6 ·
Replies
6
Views
2K
  • · Replies 11 ·
Replies
11
Views
2K
Replies
15
Views
4K
Replies
5
Views
2K
  • · Replies 3 ·
Replies
3
Views
2K
  • · Replies 2 ·
Replies
2
Views
1K